Difference between revisions of "2021 Fall AMC 10B Problems/Problem 6"

(Solution)
(Solution 2)
Line 13: Line 13:
  
 
==Solution 2==
 
==Solution 2==
 +
Recall that <math>6^k</math> can be written as <math>2^k \cdot 3^k</math>. Since we want the integer to have <math>2021</math> divisors, we must have it in the form <math>p_1^42 \cdot p_2^46</math>, where <math>p_1</math> and <math>p_2 are prime numbers. Therefore, we want </math>p_1<math> to be </math>3<math> and </math>p_2<math> to be </math>2<math>. To make up the remaining </math>2^4<math>, we multiply </math>2^42 \cdot 3^42<math> by </math>m<math>, which is </math>2^4<math> which is 16. Therefore, we have </math>42 + 16 = \boxed {(B) 58}$
 +
 +
~Arcticturn
  
 
==See Also==
 
==See Also==
 
{{AMC10 box|year=2021 Fall|ab=B|num-a=7|num-b=5}}
 
{{AMC10 box|year=2021 Fall|ab=B|num-a=7|num-b=5}}
 
{{MAA Notice}}
 
{{MAA Notice}}

Revision as of 11:52, 23 November 2021

Problem

The least positive integer with exactly $2021$ distinct positive divisors can be written in the form $m \cdot 6^k$, where $m$ and $k$ are integers and $6$ is not a divisor of $m$. What is $m+k?$

$(\textbf{A})\: 47\qquad(\textbf{B}) \: 58\qquad(\textbf{C}) \: 59\qquad(\textbf{D}) \: 88\qquad(\textbf{E}) \: 90$

Solution

Let this positive integer be written as $p_1^{e_1}\cdot p_2^{e_2}$. The number of factors of this number is therefore $(e_1+1) \cdot (e_2+1)$, and this must equal 2021. The prime factorization of 2021 is $43 \cdot 47$, so $e_1+1 = 43 \implies e_1=42$ and $e_2+1=47\implies e_2=46$. To minimize this integer, we set $p_1 = 3$ and $p_2 = 2$. Then this integer is $3^{42} \cdot 2^{46} = 2^4 \cdot 2^{42} \cdot 3^{42} = 16 \cdot 6^{42}$. Now $m=16$ and $k=42$ so $m+k = 16 + 42 = 58 = \boxed{B}$

~KingRavi

Solution 2

Recall that $6^k$ can be written as $2^k \cdot 3^k$. Since we want the integer to have $2021$ divisors, we must have it in the form $p_1^42 \cdot p_2^46$, where $p_1$ and $p_2 are prime numbers. Therefore, we want$p_1$to be$3$and$p_2$to be$2$. To make up the remaining$2^4$, we multiply$2^42 \cdot 3^42$by$m$, which is$2^4$which is 16. Therefore, we have$42 + 16 = \boxed {(B) 58}$

~Arcticturn

See Also

2021 Fall AMC 10B (ProblemsAnswer KeyResources)
Preceded by
Problem 5
Followed by
Problem 7
1 2 3 4 5 6 7 8 9 10 11 12 13 14 15 16 17 18 19 20 21 22 23 24 25
All AMC 10 Problems and Solutions

The problems on this page are copyrighted by the Mathematical Association of America's American Mathematics Competitions. AMC logo.png